Đến nội dung

hoanglong2k nội dung

Có 992 mục bởi hoanglong2k (Tìm giới hạn từ 29-04-2020)



Sắp theo                Sắp xếp  

#633549 $5^{x}.7^{y}+4=3^{z}$

Đã gửi bởi hoanglong2k on 16-05-2016 - 21:35 trong Số học

Tìm bộ 3 số $(x,y,z)$ tự nhiên thỏa mãn phương trình :  

                $5^{x}.7^{y}+4=3^{z}$

 Ta có $3^z\equiv 4\pmod 5\Rightarrow 3^z\equiv -1\pmod 5$. Nếu $z$ lẻ thì $z=2z_1+1\Rightarrow -1\equiv 3.9^{z_1}\equiv 3.(-1)^{z_1}\pmod 5$ vô lí, do đó $z$ chẵn.

 Đặt $z=2k$ với $k\in \mathbb{N}$ thì $5^x7^y=(3^k-2)(3^k+2)$, từ đó $\exists a,b,c,d\in \mathbb{N}$ sao cho $3^k-2=5^a7^b,\ 3^k+2=5^c7^d$ với $a+c=x$ và $b+d=y$

 Suy ra $5^c7^d-5^a7^b=4$ không chia hết cho $5$ và $7$, từ đó ta có các TH sau

 TH1. $c=0$, khi đó nếu $d=0$ thì $3^k+2=1$ vô lí, cho nên suy ra $b=0$, suy ra $5^a=7^d-4\equiv 0\pmod 3$ vô lí nên trường hợp này vô nghiệm

 TH2. $a=0$, khi đó nếu $b=0$ thì $5^c7^d=5$ suy ra $c=1,\ d=0$, từ đó suy ra $k=1\Rightarrow (x,y,z)=(1,0,2)$

 Nếu $d=0$ thì $5^c=4+7^b$, nếu như $b$ lẻ thì $b=2b_1+1\Rightarrow 5^c=4+7.49^{b_1}\equiv -1+7.(-1)^{b_1}\pmod 5\Rightarrow 5\mid -1+7.(-1)^{b_1}$ vô lí

 Do đó $b$ chẵn, đặt $b=2b_2\Rightarrow 5^c=4+49^{b_2}\equiv 4+(-1)^{b_2}\pmod 5$ nên $b_2$ chẵn, đặt $b_2=2b_3$ thì $5^c=2401^{b_3}+4$

 Mà $2401^{b_3}+4\equiv 5\pmod {25}$ cho nên $5^c\equiv 5\pmod {25}$. Điều này xảy ra khi và chỉ khi $c=1$, suy ra $b=0$ và $(x,y,z)=(1,0,2)$

 Vậy phương trình có nghiệm duy nhất $(x,y,z)=(1,0,2)$




#633447 $\sum \frac{a^{2}b}{1+a+b}\...

Đã gửi bởi hoanglong2k on 16-05-2016 - 14:26 trong Bất đẳng thức và cực trị

 

Bài toán:
 
Cho $a,b,c$ là các số thực không âm thỏa mãn tổng của chúng bằng 3.Chứng minh rằng:
$$(a^3b+b^3c+c^3a)(ab+bc+ca)\leq 16$$
 
 

 Đại khái là ta có $3(a^2b+b^2c+c^2a)=(a^3b+b^3c+c^3a)+(ab+bc+ca)^2-3abc$

 Cho nên $a^3b+b^3c+c^3a=3\sum a^2b+3abc-(ab+bc+ca)^2\leq 12-(ab+bc+ca)^2$

 Từ đó $(a^3b+b^3c+c^3a)(ab+bc+ca)\leq \left[12-(ab+bc+ca)^2\right](ab+bc+ca)\leq 16\Leftrightarrow (ab+bc+ca+4)(ab+bc+ca-2)^2\geq 0$

 Dấu "=" xảy ra khi $(a,b,c)=(2,1,0)$ và các hoán vị tương ứng

 




#633437 $a^{2}+b^{2}+c^{2}\geq ab+bc+ca$

Đã gửi bởi hoanglong2k on 16-05-2016 - 12:32 trong Bất đẳng thức - Cực trị

Cho $a,b,c\geq 0$, trong đó không có 2 số nào đồng thời bằng 0. Chứng minh rằng:

$a^{2}+b^{2}+c^{2}\geq^{(1)}\frac{a^{3}(b+c)}{a^{2}+bc}+\frac{b^{3}(c+a)}{b^{2}+ca}+\frac{c^{3}(a+b)}{c^{2}+ab}\geq^{(2)} ab+bc+ca$

 Cả 2 bất đẳng thức trên đều sử dụng bất đẳng thức Vornicu Schur để chứng minh :)

 Ta có $(1)\Leftrightarrow \sum a^2\left [1-\dfrac{a(b+c)}{a^2+bc}\right]\geq 0\Leftrightarrow \sum \dfrac{a^2}{a^2+bc}(a-b)(a-c)\geq 0$

 Do 2 bộ số $(a,b,c)$ và $\left (\dfrac{a^2}{a^2+bc},\dfrac{b^2}{b^2+ca},\dfrac{c^2}{c^2+ab}\right)$ đơn điệu cùng chiều nên ta có điều cần chứng minh

 Tương tự, ta có $a^2+b^2+c^2-ab-bc-ca=\sum (a-b)(a-c)$ cho nên ta sẽ viết $(2)$ về lại thành

 $\sum a^2-\sum ab\geq \sum a^2-\sum \dfrac{a^3(b+c)}{a^2+bc}$

 $\Leftrightarrow \sum (a-b)(a-c)\geq \sum \dfrac{a^2}{a^2+bc}(a-b)(a-c)$

 $\Leftrightarrow \sum \dfrac{bc}{a^2+bc}(a-b)(a-c)$

 Mặt khác 2 bộ $(a,b,c)$ và $\left (\dfrac{bc}{a^2+bc},\dfrac{ca}{b^2+ca},\dfrac{ab}{c^2+ab}\right)$ đơn điệu ngược chiều nên ta có điều cần chứng minh




#632901 $\dfrac{(a-b)(b-c)(c-a)}{abc} \leq \s...

Đã gửi bởi hoanglong2k on 13-05-2016 - 17:05 trong Bất đẳng thức - Cực trị

1. Cho các số dương $a,b,c$ và $k\geq 0$ thỏa mãn $(a+b+c)\left(\dfrac{1}{a}+\dfrac{1}{b}+\dfrac{1}{c}\right)=k+9$. Chứng minh rằng

\[\dfrac{k+6+\sqrt{k^2+36k+216-8\sqrt{(k+9)^3}}}{2}\geq \dfrac{a}{b}+\dfrac{b}{c}+\dfrac{c}{a}\geq \dfrac{k+6-\sqrt{k^2+36k+216-8\sqrt{(k+9)^3}}}{2}\]

2. Cho các số thực dương $a,b,c$ và $k\geq 0$ thỏa mãn $(a+b+c)\left (\dfrac{1}{a}+\dfrac{1}{b}+\dfrac{1}{c}\right )=k+9$. Chứng minh rằng \[\dfrac{(a-b)(b-c)(c-a)}{abc} \leq \sqrt{k^2+36k+216-8\sqrt{(k+9)^3}}\]

 

 

Bài 1

 

Bài 2

 

 




#632761 $\frac{(a^3+b^3+c^3+3abc)^2}{\sum (a^2b^2)...

Đã gửi bởi hoanglong2k on 12-05-2016 - 20:59 trong Bất đẳng thức và cực trị

Cho $a,b,c$ là các số thực dương. Tìm giá trị nhỏ nhất của biểu thức:

                                $\frac{(a^3+b^3+c^3+3abc)^2}{(a^2b^2+b^2c^2+c^2a^2)(ab+bc+ca)}$

 Ta chứng minh $\dfrac{(a^3+b^3+c^3+3abc)^2}{(a^2b^2+b^2c^2+c^2a^2)(ab+bc+ca)}\geq 4$ hay $(a^3+b^3+c^3+3abc)^2\geq 4(a^2b^2+b^2c^2+c^2a^2)(ab+bc+ca)$

 Chuẩn hóa $a+b+c=1$ và đặt $q=ab+bc+ca,\ r=abc$ thì ta cần chứng minh

$(1-3q+6r)^2\geq 4(q^2-2r)q\Leftrightarrow 36r^2+r(12-28q)-4q^3+9q^2-6q+1\geq 0$

 Với $q\leq \dfrac{1}{4}$ thì $36r^2+r(12-28q)-4q^3+9q^2-6q+1\geq -4q^3+9q^2-6q+1=(q-1)^2(1-4q)\geq 0$

 Với $\dfrac{1}{4}\leq q\leq \dfrac{1}{3}$ thì $r\geq \dfrac{(4q-1)(1-q)}{6}$ nên

$36r^2+r(12-28q)-4q^3+9q^2-6q+1\geq (4q-1)^2(1-q)^2+\dfrac{(4q-1)(1-q)(6-14q)}{3}-4q^3+9q^2-6q+1$

$=\dfrac{4q(q-1)(3q-1)(4q-1)}{3}\geq 0 $

 




#631920 ĐỀ THI OLYMPIC CHUYÊN KHOA HỌC TỰ NHIÊN 2016

Đã gửi bởi hoanglong2k on 08-05-2016 - 12:36 trong Thi HSG cấp Tỉnh, Thành phố. Olympic 30-4. Đề thi và kiểm tra đội tuyển các cấp.

Đặt $t=a+b+c$

Từ giả thiết $1\leq \frac{t^2}{3}+\frac{2t^3}{27}\rightarrow t\geq \frac{3}{2}$

$\sum \frac{a(a+1)}{(2a+1)^2}$

$=\frac{1}{4}\sum \left ( 1-\frac{1}{(2a+1)^2} \right )$

$=\frac{3}{4}-\frac{1}{4}\sum \frac{1}{(2a+1)^2}$

$\leq \frac{3}{4}-\frac{1}{12}\left ( \sum \frac{1}{2a+1} \right )^2$

$\leq \frac{3}{4}-\frac{1}{12}\left ( \frac{9}{2(a+b+c)+3} \right )^2\leq \frac{3}{4}-\frac{1}{12}.\left ( \frac{9}{6} \right )^2=\frac{9}{16}$

Dấu "=" xảy ra khi $a=b=c=1.5$

 Một hướng tiếp cận khác cho bài này như sau :

 Từ điều kiện ta suy ra tồn tại các số dương $x,y,z$ sao cho $a=\dfrac{x}{y+z}\, \cdots $ tương tự cho $b$ và $c$

 Viết lại bất đẳng thức thành $\sum \dfrac{x(x+y+z)}{(2x+y+z)^2}\leq \dfrac{9}{16}$

 Mặt khác ta có $(x+y+z)\sum \dfrac{x}{(2x+y+z)^2}\leq (x+y+z)\sum \dfrac{x}{4(x+y)(x+z)}=\dfrac{1}{2}.\dfrac{(xy+yz+zx)(x+y+z)}{(x+y)(y+z)(z+x)}\leq \dfrac{9}{16}$




#630285 $\sum \frac{1}{4a^{2}-ab+4b^{2...

Đã gửi bởi hoanglong2k on 30-04-2016 - 05:28 trong Bất đẳng thức - Cực trị

cho a,b,c không âm 

$\sum \frac{1}{4a^{2}-ab+4b^{2}}\geq \frac{9}{7(a^{2}+b^{2}+c^{2})}$

 Áp dụng bất đẳng thức Cauchy-Schwarz ta có 

 $\sum \dfrac{1}{4a^2-ab+4c^2}=\sum \dfrac{(2a+2b+5c)^2}{(4a^2-ab+4b^2)(2a+2b+5c)^2}\geq \dfrac{81(a+b+c)^2}{\sum (4a^2-ab+4b^2)(2a+2b+5c)^2}$

 Nên ta chỉ cần chứng minh $63(a+b+c)^2(a^2+b^2+c^2)\geq \sum (4a^2-ab+4b^2)(2a+2b+5c)^2$

 Hay là $31\sum a^4+18\sum ab(a^2+b^2)+31abc\sum a\geq 98\sum a^2b^2$

 Áp dụng bất đẳng thức Schur bậc 4 ta có $\sum a^4+abc\sum a\geq \sum ab(a^2+b^2)$

 Từ đó kết hợp với AM-GM thì $31\sum a^4+18\sum ab(a^2+b^2)+31abc\sum a\geq 49\sum ab(a^2+b^2)\geq 98\sum a^2b^2$

 Bài toán được chứng minh. Dấu "=" xảy ra khi $a=b=c$ hoặc $a=b,\ c=0$ và các hoán vị 

 

 




#629699 $\frac{a}{\sqrt{ka^{2}+b^{2...

Đã gửi bởi hoanglong2k on 26-04-2016 - 20:05 trong Bất đẳng thức và cực trị

Thay $k=7$ thôi. Số đẹp lắm !!!

 Khi em thay $k=4$ thì bài này xuất hiện trong đề thi HSG lớp 11 tỉnh Quảng Bình năm 2014-2015 :) Chứng minh $P(4)\leq \sqrt{\dfrac{3}{4}}$




#628510 $\frac{a}{\sqrt{ka^{2}+b^{2...

Đã gửi bởi hoanglong2k on 20-04-2016 - 17:15 trong Bất đẳng thức và cực trị

Em cũng dùng $Cauchy-Schwarz$ giống anh mà cách em hơi kì cục tí: 

$(ka^{2}+b^{2}+c^{2})(k+1+1)\geq (ka+b+c)^{2}\Rightarrow ka^{2}+b^{2}+c^{2}\geq \frac{(ka+b+c)^{2}}{k+2}\Rightarrow \frac{a}{\sqrt{ka^{2}+b^{2}+c^{2}}}\leq \frac{a\sqrt{k+2}}{ka+b+c}.$ Tương tự suy ra:

$\sum \dfrac{a}{\sqrt{ka^2+b^2+c^2}}\leq \sum \frac{a\sqrt{k+2}}{ka+b+c}.$

$\frac{1}{ka+b+c}=\frac{1}{\frac{k-1}{2}a+\frac{k-1}{2}a+a+b+c}\leq \frac{1}{9}(\frac{1}{\frac{k-1}{2}a}+\frac{1}{\frac{k-1}{2}a}+\frac{1}{a+b+c})=\frac{1}{9}(\frac{4}{(k-1)a}+\frac{1}{a+b+c})\Rightarrow \frac{a}{\sqrt{ka^{2}+b^{2}+c^{2}}}\leq \frac{a\sqrt{k+2}}{9}(\frac{4}{(k-1)a}+\frac{1}{a+b+c})=\frac{4}{9}.\frac{\sqrt{k+2}}{k-1}+\frac{a\sqrt{k+2}}{9(a+b+c)}.$

Tương tự, cộng lại có điều phải chứng minh. 

------------------

Anh xem vậy có được không ạ???

 À ừ được rồi em, em dò mà không thấy sai thì thôi, sao lại đi hỏi anh làm gì =))

 Thực ra thì lúc đầu anh tưởng đây là một bài toán tổng quát cho số thực dương $k>1$ bất kì, nhưng trong bài làm thì em lại cho $k=7$ mất, có nghĩa là tại một giá trị $k\neq 7$ mà đề ra tìm giá trị lớn nhất thì bài toán của em không ổn  :unsure: và bài toán của em theo trên thì nó thành 4 biến mất rồi :P 

 Nhưng cũng chúc mừng em vì sáng tác được bài toán mớ :)  :B)




#628461 $\frac{a}{\sqrt{ka^{2}+b^{2...

Đã gửi bởi hoanglong2k on 20-04-2016 - 12:37 trong Bất đẳng thức và cực trị

Chứng minh với mọi $a,b,c$ dương và $k>1$ thì ta có bất đẳng thức sau: 

$\frac{a}{\sqrt{ka^{2}+b^{2}+c^{2}}}+\frac{b}{\sqrt{a^{2}+kb^{2}+c^{2}}}+\frac{c}{\sqrt{a^{2}+b^{2}+kc^{2}}}\leq \frac{\sqrt{k+2}}{3}(\frac{4}{k-1}+\frac{1}{3}).$

P/s: Đây là một bài bất đẳng thức mình chế lại, mọi người xem thử :))).

 Áp dụng bất đẳng thức Cauchy-Schwarz ta có $\sum \dfrac{a}{\sqrt{ka^2+b^2+c^2}}\leq \sqrt{3\sum \dfrac{a^2}{ka^2+b^2+c^2}}$

 Mặt khác ta lại có $\dfrac{1}{a^2+b^2+c^2}+\dfrac{\left (\dfrac{k-1}{3}\right )^2}{(k-1)a^2}\geq \dfrac{\left (1+\dfrac{k-1}{3}\right )^2}{ka^2+b^2+c^2}$

 Hay là $\dfrac{1}{a^2+b^2+c^2}+\dfrac{k-1}{9a^2}\geq \dfrac{(k+2)^2}{9(ka^2+b^2+c^2)}$

 Từ đó thì $\sum \dfrac{a^2}{ka^2+b^2+c^2}\leq \dfrac{9}{(k+2)^2}\sum \left (\dfrac{a^2}{a^2+b^2+c^2}+\dfrac{k-1}{9}\right )=\dfrac{9}{(k+2)^2}\left (1+\dfrac{k-1}{3}\right )=\dfrac{3}{k+2}$

 Đến đây anh nghĩ cho kết thúc là đủ vì có thể kết luận được GTLN cho biểu thức nếu bài toán là tìm GTLN

 Còn nếu không thì ta biến đổi tương được $(k-7)^2\geq 0$

 Dấu "=" cho cả bài là khi $a=b=c$ và $k=7$ :)

 




#627113 $\sum \left (\dfrac{a}{3a^2+abc+27}...

Đã gửi bởi hoanglong2k on 14-04-2016 - 19:38 trong Bất đẳng thức - Cực trị

Tìm hằng số $k$ lớn nhất sao cho bất đẳng thức sau luôn đúng với mọi số thực $a,b,c$ dương có tổng bằng 3

\[\left (\dfrac{a}{3a^2+abc+27}\right )^k+\left (\dfrac{b}{3b^2+abc+27}\right )^k+\left (\dfrac{c}{3c^2+abc+27}\right )^k\leq \dfrac{3}{31^k}\]




#626813 CMR: $\frac{2(a^5+b^5+c^5+a^2+b^2+c^2)}{a^3+b^3+c^3...

Đã gửi bởi hoanglong2k on 12-04-2016 - 19:53 trong Bất đẳng thức và cực trị

Cho $a,b,c>0$. Chứng minh rằng

                                $\frac{2(a^5+b^5+c^5+a^2+b^2+c^2)}{a^3+b^3+c^3}+\frac{9}{(a+b+c)^2}\geq 5$

 Áp dụng bất đẳng thức AM-GM ta có $a^5+a^2+a^2\geq 3a^3\Rightarrow \sum a^5+2\sum a^2\geq 3\sum a^3$

 Áp dụng bất đẳng thức Chebyshev ta có $a^5+b^5+c^5\geq \dfrac{1}{3}(a^3+b^3+c^3)(a^2+b^2+c^2)$

 Suy ra : 

$2(a^5+b^5+c^5+a^2+b^2+c^2)\geq \left ( 3+\dfrac{a^2+b^2+c^2}{3} \right )(a^3+b^3+c^3)$

$\Rightarrow \frac{2(a^5+b^5+c^5+a^2+b^2+c^2)}{a^3+b^3+c^3}\geq 3+\dfrac{a^2+b^2+c^2}{3}\geq 3+\dfrac{(a+b+c)^2}{9}$

 Nên ta chỉ cần chứng minh $\dfrac{(a+b+c)^2}{9}+\dfrac{9}{(a+b+c)^2}\geq 2$

 Đây chính là bất đẳng thức AM-GM :)

 Dấu "=" xảy ra khi $a=b=c=1$




#623820 Chứng minh $MI // DA$

Đã gửi bởi hoanglong2k on 31-03-2016 - 12:52 trong Hình học

đề của anh ego đúng đấy
thực ra sau khi sử dụng phương tích ta sẽ đưa về bài toán quen thuộc sau đây
cho tam giác ABC trên AB, AC lấy E,F sao cho BE bằng CF gọi I, M là trung điểm EF và BC thì lúc này ta có IM song song với tia phân giác góc a
tính chất này có thể xem lại ở topic mỗi tuần một bài toán, em cũng đã từng sử dụng tính chất này ở đó

 Cùng ý tưởng  :like

 Đây là lời giải của mình

 Gọi $J$ là giao điểm của $AD$ với $(ABC)$ 

 Gọi $O$ là tâm $(ABC)$ và $L$ là tâm $(AEF)$. Sử dụng phương tích ta có $\dfrac{BD.BM}{CM.CD}=\dfrac{BE.BA}{CF.CA}\Leftrightarrow BE=CF$

 Khi đó, theo một tính chất quen thuộc thì $(O)$ và $(L)$ cắt nhau tại $K$ là điểm chính giữa cung $BC$ khác $J$, khi đó $KJ$ là đường kính nên $AD\perp AK$

 Mà $AK$ là trục đẳng phương của $(O)$ và $(L)$ nên $OL\perp AK$

 Mặt khác thì $\Delta KEF\sim \Delta KBC$ nên $\dfrac{KO}{KM}=\dfrac{KL}{KI}$ nên $LO//IM$

 Từ đó $IM//AD$

Hình gửi kèm

  • Untitled.png



#622403 Đề thi HSG lớp 11 tỉnh Quảng Bình 2 ngày

Đã gửi bởi hoanglong2k on 24-03-2016 - 21:57 trong Thi HSG cấp Tỉnh, Thành phố. Olympic 30-4. Đề thi và kiểm tra đội tuyển các cấp.

 Lời giải của mình cho bài 5 ngày 2.

 Với $n=1$ thì bài toán thỏa mãn

 Với $n\geq 2$, do $2^{2n-1}-2^n+1$ lẻ nên để nó là số chính phương thì phải là số chính phương lẻ. Đặt $2^{2n-1}-2^n+1=(2k+1)^2$ với $k\in \mathbb{N^*}$

 Khi đó phương trình tương đương với $2^n(2^{n-1}-1)=4k(k+1)$

 Ta xét 2 trường hợp nhỏ

    - TH1: Nếu $k$ chẵn, đặt $k=2^r.s$ với $(s,2)=1$, $r,s\in \mathbb{N^*}$, phương trình tương đương $2^n(2^{n-1}-1)=2^{r+2}s(2^r.s+1)$

    Khi đó, từ phương trình ta sẽ suy ra $2^n=2^{r+2}$ và $2^{n-1}-1=s(2^rs+1)$

    Từ $2^n=2^{r+2}$ suy ra $2^n=2^{r+2}=0$ hoặc $n=r+2$ mà $2^n>0$ nên $n=r+2$

    Suy ra $2^{r+1}-1=2^r.s^2+s$ hay $2^r(2-s^2)=s+1$ suy ra $s^2<2\Rightarrow s=1\Rightarrow r=1\Rightarrow n=3$

    - TH2: Nếu $k$ lẻ thì $k+1$ chẵn. Đặt $k+1=2^u.v$ với $(v,2)=1,\ v,s\in \mathbb{N^*}$

    Khi đó phương trình tương đương $2^n(2^{n-1}-1)=4(2^uv-1).2^uv$, tương tự trường hợp trên ta suy ra $n=u+2$ và $2^{n-1}-1=(2^uv-1)v$

    Hay $2^{u+1}-1=2^uv^2-v\Leftrightarrow v-1=2^u(v^2-2)\Rightarrow v^2-2\mid v-1\Rightarrow v-1\geq v^2-2\Rightarrow v=1$

    Mà từ đó thì $2^{u+1}=2^u$ điều này là vô lí. Vậy trong TH này không có $n$ thỏa mãn

  Vậy $n=1$ và $n=3$ là hai số thỏa mãn yêu cầu bài toán :)




#621564 $\dfrac{a-b}{ab+4b+}+\dfrac{b-c}...

Đã gửi bởi hoanglong2k on 20-03-2016 - 22:57 trong Bất đẳng thức - Cực trị

Cho $a,b,c$ là các số thực không âm. Chứng minh rằng \[\dfrac{a-b}{ab+4b+4}+\dfrac{b-c}{bc+4c+4}+\dfrac{c-a}{ca+4a+4}\geq 0\] 




#621325 Violympic 2015

Đã gửi bởi hoanglong2k on 20-03-2016 - 04:15 trong Góc giao lưu

Mới thế mà đã một năm rồi :-D Nhớ quá cái hồi đó :)




#621324 Người yêu toán bình Tam Quốc

Đã gửi bởi hoanglong2k on 20-03-2016 - 02:45 trong Quán văn

Theo mình thì Gia Cát Lượng hiện lên trong phim không khác gì một con yêu quái. Cái gì ông ta cũng biết, cái gì cũng tính được trước. 

 

Lưu bị (theo ý viết của La Quán Trung) là nhân vật chính diện, nhưng lại không được ai yêu. Tào Tháo là nhân vật phản diện nhưng lại được nhiều người thích. Tuy nhiên, thiên hạ lại nhiều người thờ Quan Vũ. Tượng ông cũng được tạc nhiều nhất. Mọi người có biết vì sao không?

 Chắc là vì sự oai hùng của ông í ạ :)




#620860 Tìm số $k$ nhỏ nhất

Đã gửi bởi hoanglong2k on 18-03-2016 - 00:46 trong Bất đẳng thức - Cực trị

 

Tìm hằng số $k$ nhỏ nhất sao cho bất đẳng thức sau đúng với mọi số thực không âm $a,b,c$:

\[abc+2+k\left [ (a-1)^2+(b-1)^2+(c-1)^2 \right ]\geq a+b+c\]

 

 Cho $a=0$ thì ta có $k\left [1+(b-1)^2+(c-1)^2\right ]\geq b+c-2$

 Áp dụng AM-GM thì $1+(b-1)^2+(c-1)^2\geq 1+\dfrac{(b+c-2)^2}{2}\geq \sqrt{2}(b+c-2)$ nên từ đó thì $k_{min}=\dfrac{1}{\sqrt{2}}$

 Việc còn lại của chúng ta là chứng minh $abc+2+\dfrac{(a-1)^2+(b-1)^2+(c-1)^2}{\sqrt{2}}\geq a+b+c$

 Áp dụng AM-GM thì $(a-1)^2+(b-1)^2+(c-1)^2\geq (1-a)^2+\dfrac{(b+c-2)^2}{2}\geq \sqrt{2}(b+c-2)(1-a)$

 Nên ta chỉ cần chứng minh $abc+2+(b+c-2)(1-a)\geq a+b+c\Leftrightarrow a(b-1)(c-1)\geq 0$

 Theo nguyên lí Dirichlet thì tồn tại ít nhất hai trong ba số $a-1,\ b-1,\ c-1$ cùng dấu nên ta có thể giả sử $(b-1)(c-1)\geq 0$

 Cho nên bất đẳng thức trên đúng

 Vậy ta có điều cần chứng minh :)




#620265 $\sum \frac{1}{a^2+1}\geq \frac{3}{2}$

Đã gửi bởi hoanglong2k on 14-03-2016 - 20:36 trong Bất đẳng thức và cực trị

Giả sử $c=\min\{a,b,c\}$ thì $ab \geqslant 1$ khi đó áp dụng bất đẳng thức quen thuộc

\[\frac{1}{a^2+1}+\frac{1}{b^2+1} \geqslant \frac{2}{ab+1},\]

ta được bài toán về chứng minh

\[\frac{2}{ab+1}+\frac{1}{c^2+1} \geqslant \frac{3}{2},\]

 Em nghĩ đến chỗ này làm thế này sẽ gọn hơn ạ :)

 Bất đẳng thức trên tương đương với $2(2c^2+ab+3)\geq 3(ab+1)(c^2+1)\Leftrightarrow c^2+3\geq 3abc^2+ab\Leftrightarrow a+b+c\geq 3abc$

 Áp dụng AM-GM thì $3(a+b+c)=(a+b+c)(ab+bc+ca)\geq 9abc$ nên ta có điều cần chứng minh

 Em cũng biết tư tưởng là đồng bậc hóa rồi AM-GM nhưng việc đồng bậc sau có lẽ sẽ gọn gàng hơn phần nào :D




#619994 $\sum \frac{1}{a^2+1}\geq \frac{3}{2}$

Đã gửi bởi hoanglong2k on 13-03-2016 - 10:39 trong Bất đẳng thức và cực trị

Ta có bất đẳng thức tương đương $\dfrac{a^2}{a^2+1}+\dfrac{b^2}{b^2+1}+\dfrac{c^2}{c^2+1}\leq \dfrac{3}{2}\Leftrightarrow \sum \dfrac{a^2}{a(a+b+c)+2a^2+bc}\leq \dfrac{1}{2}$

Áp dụng bất đẳng thức Cauchy-Schwarz thì $\sum \dfrac{a^2}{a(a+b+c)+2a^2+bc}\leq \sum \dfrac{a^2}{4}\left (\dfrac{1}{a(a+b+c)}+\dfrac{1}{2a^2+bc}\right )=\dfrac{1}{4}+\sum \dfrac{a^2}{4(2a^2+bc)}$

Nên ta chỉ cần chứng minh $\sum \dfrac{a^2}{2a^2+bc}\leq 1$ hay $\sum \dfrac{bc}{2a^2+bc}\geq 1$

Áp dụng bất đẳng thức Cauchy-Schwarz thì $\sum \dfrac{(bc)^2}{2a^2bc+b^2c^2}\geq \dfrac{(ab+bc+ca)^2}{a^2b^2+b^2c^2+c^2a^2+2abc(a+b+c)}=1$

Nên ta có điều cần chứng minh




#617907 $\frac{x^7 - 1}{x - 1} = y^5 - 1$

Đã gửi bởi hoanglong2k on 01-03-2016 - 20:55 trong Số học

 Câu 2. Do $N$ chẵn nên $N$ có dạng $2^k.r$ với $k\geq 1$ và $r$ lẻ.

Mặt khác $N$ là số hoàn hảo nên  $2N=\sigma (N)\Leftrightarrow 2^{k+1}m=\sigma(2^k).\sigma(m)=\left ( 2^{k+1}-1 \right )\sigma(m)\Rightarrow \sigma(m)=m+\dfrac{m}{2^{k+1}-1}\Rightarrow 2^{k+1}-1|m$

 Nếu $m=2^{k+1}-1$ thì ta có $N=2^k(2^{k+1}-1)=\dfrac{2^{k+1}(2^{k+1}-1)}{2}$ là số hoàn hảo

 Nêu $m=(2^{k+1}-1)l$ với $l\geq 2$ thì $l|m;1|m;m|m$, khi đó $m+l=\sigma (m)\geq m+l+1$ vô lí

 Vậy ta có điều cần chứng minh




#617007 [Trường Xuân toán học miền nam 2016] Vietnam TST 2016 MOCK Test 2

Đã gửi bởi hoanglong2k on 26-02-2016 - 13:22 trong Thi HSG cấp Tỉnh, Thành phố. Olympic 30-4. Đề thi và kiểm tra đội tuyển các cấp.

 Bài 5. 

 Gọi $A_3$ là giao điểm $AA_1$ với $(O)$

 Khi đó $AA_2$ là đường đối trung của tam giác $ABC$ nên $\Delta ABA_2\sim \Delta AA_3C\Rightarrow AA_2.AA_3=AB.AC$

 Mặt khác sử dụng đẳng thức Ptolemy ta có $A_1A.A_1A_3=A_1B.A_1C\Leftrightarrow AA_1(AA_3-AA_1)=\dfrac{BC^2}{4}\Rightarrow AA_1.AA_3=\dfrac{AB^2+AC^2}{2}$

 Từ đó suy ra $\dfrac{AA_2}{AA_1}=\dfrac{2AB.AC}{AB^2+AC^2}$

 Đặt $AB=c;BC=a;CA=b$ thì ta có $\dfrac{AA_2}{AA_1}=\dfrac{2bc}{b^2+c^2}$, tương tư cho các phân thức kia

 Mà theo công thức tính diện tích tam giác ta lại có $S=\dfrac{abc}{4R}=pr$

$\Rightarrow S^2=\dfrac{abcpr}{4R}\Rightarrow \dfrac{4r}{R}=\dfrac{16S^2}{abcp}=\dfrac{16p(p-a)(p-b)(p-c)}{abcp}=\dfrac{2(a+b-c)(b+c-a)(c+a-b)}{abc}$

 Vì thế nên ta chỉ cần chứng minh $\dfrac{2bc}{b^2+c^2}+\dfrac{2ca}{c^2+a^2}+\dfrac{2ab}{a^2+b^2}\geq 1+\dfrac{2(a+b-c)(b+c-a)(c+a-b)}{abc}$

 Bất đẳng thức tương đương với :

$\sum (b-c)^2\left ( \dfrac{b+c-a}{abc}-\dfrac{1}{b^2+c^2} \right )\geq 0\Leftrightarrow S_a(b-c)^2+S_b(c-a)^2+S_c(a-b)^2\geq 0$ trong đó $S_a=\dfrac{b+c-a}{abc}-\dfrac{1}{b^2+c^2}$, ...

 Giả sử $a\geq b\geq c$, khi đó dễ dàng chứng minh được $S_b\geq 0$ và $S_b+S_c\geq 0$

 Mặt khác, do $a,b,c$ là 3 cạnh của một tam giác nên ta cũng chứng minh $S_a+S_b\geq 0$

 Khi đó theo tiêu chuẩn SOS ta có điều cần chứng minh

 Dấu "=" xảy ra khi và chỉ khi $a=b=c$ hay $\Delta ABC$ là tam giác đều :)




#616742 [Trường Xuân toán miền Nam]Vietnam TST 2016 Mock Test 1

Đã gửi bởi hoanglong2k on 24-02-2016 - 20:01 trong Thi HSG cấp Tỉnh, Thành phố. Olympic 30-4. Đề thi và kiểm tra đội tuyển các cấp.

Mình xin làm bài bất đẳng thức

Đặt $a+b+c=p , ab+bc+ca=q , abc=r $

Ta cần chứng minh bđt 

$\frac{q+\sqrt{q^2-2pr}}{p+\sqrt{p^2-2q}} \geq \frac{p}{3} $

Bằng cách bình phương 2 vế, ta đưa được về dạng $f(r) \geq 0$

Mà ta có $f'(r) \geq 0 $ nên theo định lý $PQR$, ta chỉ cần chứng minh bđt trong trường hợp có $2$ số bằng nhau 

Không mất tính tổng quát, giả sử $a=b $

Chuẩn hóa $abc=1 => c=\frac{1}{a^2} $

BĐT trở thành

$f(a)=a^2-\frac{2}{a} +\frac{1}{a^4} + 2\sqrt{2a^4+1} +\sqrt{\frac{2}{a^2}+ \frac{1}{a^6}}-3\sqrt{a^4+\frac{2}{a^2}} \geq 0 $

Khảo sát hàm 1 biến, ta thấy $f(a) \geq f(1) = 0$

Do đó, ta có điều phải chứng minh

Dấu bằng xảy ra khi $a=b=c $

Cách này hơi trâu

Mong chờ một lời giải đẹp từ các bạn 

 Bài này cho đơn giản thì chuẩn hóa $p=3$ sau đó dùng bất đẳng thức Schur $r\geq \dfrac{4q-9}{3}$ sẽ đơn giản hơn nhiều 

 Ta có bất đẳng thức tương đương $\dfrac{q+\sqrt{q^2-2pr}}{p+\sqrt{p^2-2q}}\leq \dfrac{p}{3}$

 Chuẩn hóa $p=3$ thì cần chứng minh $q+\sqrt{q^2-6r}\leq 3+\sqrt{9-2q}$

 Áp dụng bất đẳng thức Schur thì $r\geq \dfrac{4q-9}{3}$, ta cần chứng minh $q+\sqrt{q^2-8q+18}\leq 3+\sqrt{9-2q}$

 Đây là bất đẳng thức một biến, xét đạo hàm sẽ ra. Hoặc biến đổi tương đương cho đỡ ( chúa ghét đạo hàm chứa căn -_- )

 Bất đẳng thức cần chứng minh tương đương với $(3-q).\left (1-\dfrac{3-q}{\sqrt{q^2-8q+18}+\sqrt{9-2q}}\right )\geq 0$

 Luôn đúng do $3\geq q$ và $\sqrt{q^2-8q+18}>\sqrt{q^2-6q+9}\geq 3-q$

 Dấu "=" khi $a=b=c$




#616359 $\sum \frac{a^2}{3a^2+(b+c)^2} \leq...

Đã gửi bởi hoanglong2k on 21-02-2016 - 23:32 trong Bất đẳng thức và cực trị

Cho $a,b,c$ là các số thực không âm sao cho $a+b+c > 0$. Chứng minh rằng:

 

$\sum \frac{a^2}{3a^2+(b+c)^2} \leq \frac{1}{2}$

 

P/s: Mong mọi người giải theo hướng Cauchy-Schwarz ạ.

 Có gì mai lên lớp anh tìm lời giải bằng Cauchy-Schwarz sau =)) Giờ bận làm công nghệ mai thuyết trình nên gõ tạm lời giải này đã

Chuẩn hóa $a+b+c=2$ thì ta có bất đẳng thức tương đương $\sum \dfrac{a^2}{3a^2+(2-a)^2}\leq \dfrac{1}{2}$

Đến đây chú ý là với hàm $f(x)=\dfrac{x^2}{4x^2-4x+4}$ với $x\in [0;2]$ thì có $f’’(x)=\dfrac{x^3-3x^2+1}{2(x^2-x+1)^3}< 0$ với $\forall x\in [0;2]$ nên $f(x)$ lồi trên $[0;2]$, khi đó $f(x)\leq \dfrac{f(0)-f(1)}{0-1}.(x-0)+f(0)=\dfrac{f(0)-f(1)}{0-1}.(x-1)+f(1)=\dfrac{x}{4}$

Nói cách khác, ta sẽ chứng minh $f(x)\leq \dfrac{x}{4}\Leftrightarrow 4x(x-1)^2\geq 0$

Thay $x$ lần lượt bởi $a,b,c$ ta sẽ có $\sum \dfrac{a^2}{3a^2+(2-a)^2}\leq \dfrac{a+b+c}{4}=\dfrac{1}{2}$

Vậy ta có điều cần chứng minh

Dấu “=” xảy ra khi và chỉ khi $(a,b,c)\sim (1,1,0)$ cùng các hoán vị




#614686 CMR: $\frac{a+3}{(a+1)^2}+\frac{b+3...

Đã gửi bởi hoanglong2k on 13-02-2016 - 11:35 trong Bất đẳng thức và cực trị

2.BĐT tương đương:

$\sum b^2c^2\geq a^2b^2c^2(a^2+b^2+c^2)$

Mặt khác: $\sum b^2c^2 \geq abc(a+b+c)$ nên chỉ cần chứng minh $a+b+c\geq abc(a^2+b^2+c^2)$

$<=>(a+b+c)+2(ab+bc+ca)abc\geq 9abc$

Sử dụng AM-GM 3 số ta được:

$(a+b+c)+2(ab+bc+ca)abc\geq 3.\sqrt[3]{(a+b+c)(ab+bc+ca)^2a^2b^2c^2}\geq 3.\sqrt[3]{27a^3b^3c^3}=9abc$

Vậy BĐT được chứng minh hoàn toàn

 Có thể chứng minh như sau : 

 Ta có $\dfrac{1}{a^2}+\dfrac{1}{b^2}+\dfrac{1}{c^2}\geq \dfrac{1}{ab}+\dfrac{1}{bc}+\dfrac{1}{ca}=\dfrac{9}{abc(a+b+c)}\geq \dfrac{27}{(ab+bc+ca)^2}$

 Nên ta chỉ cần chứng minh $(a^2+b^2+c^2)(ab+bc+ca)^2\leq 27=\dfrac{(a+b+c)^6}{27}$ đúng theo AM-GM.

 

3.Cho $a,b,c$ là các số thực dương

CMR:  $\frac{(b+c-a)^2}{2a^2+(b+c)^2}+\frac{(c+a-b)^2}{2b^2+(c+a)^2}+\frac{(a+b-c)^2}{2c^2+(a+b)^2} \geq \frac{3}{2}.\frac{a^2+b^2+c^2}{(a+b+c)^2}$

 Ta có $\sum \dfrac{(b-c+a)^2}{2a^2+(b+c)^2}\geq \sum \dfrac{(b-c+a)^2}{2(a^2+b^2+c^2)}=\dfrac{3(a^2+b^2+c^2)-2(ab+bc+ca)}{2(a^2+b^2+c^2)}$

 Nên ta chỉ cần chứng minh

 $\dfrac{3(a^2+b^2+c^2)-2(ab+bc+ca)}{2(a^2+b^2+c^2)}\geq \dfrac{3(a^2+b^2+c^2)}{2[a^2+b^2+c^2+2(ab+bc+ca)]}\Leftrightarrow 8(ab+bc+ca)(a^2+b^2+c^2-ab-bc-ca)\geq 0$